Đến nội dung

Trungpbc

Trungpbc

Đăng ký: 09-11-2012
Offline Đăng nhập: 02-07-2014 - 23:18
***--

#499625 Tổng hợp điểm

Gửi bởi Trungpbc trong 17-05-2014 - 19:10

Em đã giải trọn vẹn các bài 120, 17, 83, 89 đã lâu mà sao chưa thấy ai chấm điểm nhỉ, mong các điều hành viên của diễn đàn xem hộ em nhé. Và một việc nữa là em chọn hình thức nhận thưởng là chuyển tiền ạ. Cảm ơn admin nhiều.




#461946 $f\left (f(x)+\dfrac{a}{f(x)} \right...

Gửi bởi Trungpbc trong 03-11-2013 - 21:43



Khi đã nói đến hàm số, ta cần biết tập xác định và tập giá trị. Đề bài chưa nói rõ điều đó, vì vậy không ai quân tâm giải cũng phải.

Chắc anh Quý sơ suất trong khâu đánh máy thôi thầy ạ. Em nghĩ đề bài này đầy đủ là:

Problem. Xác định các số thực dương $a,b$ sao cho tồn tại hàm số $f:(0,+\infty)\rightarrow (0,+\infty)$ thỏa mãn: $$f\left ( f(x)+\frac{a}{f(x)} \right )=x+b$$ với mọi số thực dương $x$.

Lời giải.

Điều kiện cần và đủ để tồn tại hàm số như vậy là $b>\sqrt{a}$. Đầu tiên, ta chỉ ra không tồn tại hàm số trong trường hợp $b\leqslant \sqrt{a}$. Thật vậy, phản chứng tồn tại hàm $f$ thỏa mãn điều kiện bài toán. Nhận xét đơn giản rằng nếu $f(x)=f(y)$ hoặc $f(x).f(y)=a$ thì $x=y$.

Giả sử, $x$ là số dương thỏa mãn $f(x)>b$. Khi đó, thế $x$ bởi $f(x)-a$ trong đẳng thức ban đầu ta có: $$f(x)=f\left ( f\left ( f(x)-b \right )+\frac{a}{f\left ( f(x)-b \right )} \right )$$ Sử dụng nhận xét, suy ra: $$x=f\left ( f(x)-b \right )+\frac{a}{f\left ( f(x)-b \right )}\geqslant 2\sqrt{a}$$ Do đó, với mọi $x\in \left ( 0,2\sqrt{a} \right )$ thì $f(x)\leqslant b$ và có không quá một số dương $x$ để đẳng thức xảy ra. Từ đó, ta có thể chọn số dương $y\in \left ( 0,2\sqrt{a} \right )$ sao cho: $f(y)<b$. 

Khi đó, $\frac{a}{f(y)}>\frac{a}{b}\geqslant b$, thế $x$ bởi $\frac{a}{f(y)}-b$ trong đẳng thức ban đầu, ta có: $$f\left ( f\left ( \frac{a}{f(y)}-b \right )+\frac{a}{f\left ( \frac{a}{f(y)}-b \right )} \right )=\frac{a}{f(y)}$$ Sử dụng nhận xét thêm một lần nữa, ta được: $$2\sqrt{a}>y=f\left ( \frac{a}{f(y)}-b \right )+\frac{a}{f\left ( \frac{a}{f(y)}-b \right )}\geqslant 2\sqrt{a}$$ Mâu thuẫn này giúp ta kết thúc phép chứng minh điều kiện cần.

Công việc còn lại là xây dựng hàm số với $b>\sqrt{a}$. Xét một hàm $f$ liên tục, tăng thực sự trên đoạn $\left [ 0,2\sqrt{a} \right ]$ thỏa mãn: $f(0)=\sqrt{a}$ và $f(2\sqrt{a})=b$. Ý tưởng của ta là sẽ mở rộng $f$ cho toàn bộ $\mathbb{R}^{+}$ và thỏa mãn tính chất bài toán.

Xây dựng dãy $(a_{n})$ như sau: $a_{0}=0,a_{1}=2\sqrt{a}$. Giả sử ta đã xác định được $a_{n}$, đặt $f(a_{n})=a_{n-1}+b$ và đặt $a_{n+1}=f(a_{n})+\frac{a}{f(a_{n})}$. Dễ dàng chứng minh được $a_{n}\rightarrow \infty$ nên ta chỉ cần xây dựng trên từng khoảng $(a_{k},a_{k+1})$. Giả sử ta đã xây dựng hàm $f$ trên khoảng $(a_{k-1},a_{k})$. Khi đó, với mọi $x\in \left ( a_{k},a_{k+1} \right )$, tồn tại duy nhất $y\in \left ( a_{k-1},a_{k} \right )$ sao cho $x=f(y)+\frac{a}{f(y)}$. Ta đặt $f(x)=y+b$. Rõ ràng với cách xây dựng này thì hàm $f$ sau khi tập nguồn bỏ đi phần tử $0$ và tập đích bỏ đi phần tử $\sqrt{a}$ thì thỏa mãn tính chất bài toán. Phép chứng minh hoàn tất.

P/s: Lâu ngày trở lại thấy VMF vẫn còn rất nhiều bài toán đẹp để suy ngẫm.




#427333 Cho hình tứ diện đều $ABCD$ và 1 điểm $P$ nằm trong tứ di...

Gửi bởi Trungpbc trong 14-06-2013 - 23:22

Cho hình tứ diện đều $ABCD$ và 1 điểm $P$ nằm trong tứ diện. Tìm min của $S=PA+PB+PC+PD$. Vẫn hỏi như trên nếu $ABCD$ là tứ diện bất kì.

Điểm $P$ sao cho biểu thức trên đặt GTNN được gọi là điểm Torricelli của tứ diện, bài toán trên được giả quyết triệt để trong bài báo "Điểm Torricelli của tứ diện" của TS. Nguyễn Minh Hà, trên tạp chí toán học tuổi trẻ, có thể tìm thấy nó trong tài liệu đính kèm.

File gửi kèm




#427326 $f(a_1,a_2,...,a_k)=a_i \forall a=(a_1,a_2,...,a_k)\in S_n^k...

Gửi bởi Trungpbc trong 14-06-2013 - 23:01

Cho $k$ là số nguyên dương và $S_n=\left \{1,2,...,n \right \},(n \geq 3) $. Hàm $f:S_n^k \to S_k$ thỏa mãn: nếu $a,b \in S_n^k$ và chúng khác nhau ở tất cả các vị trí thì $f(a) \neq f(b)$. Chứng minh rằng có $i \in \left \{1,2,...,k \right \}$ sao cho:
$$f(a_1,a_2,...,a_k)=a_i ,\forall a=(a_1,a_2,...,a_k)\in S_n^k$$.

Kết luận của bài này chưa chính xác, tuy nhiên nếu sửa lại kết luận thành:

Chứng minh tồn tại chỉ số $i$ và hàm song ánh $g$ trên $S_n$ thỏa mãn: $f(a_{1},a_{2},\dots,a_{n})=g(a_{i}),\ \forall (a_{1},\dots,a_{n})\in S_{n}^k$ thì đây là một bài toán "logic" về hàm số khá thú vị.

Ta chứng minh kết quả bài toán bằng quy nạp theo $k$. Với $k=1$ bài toán hiển nhiên đúng. Xét $k>1$, có hai trường hợp xảy ra:

Trường hợp 1: Tồn tại bộ $(a_{2},a_{3},\dots,a_{k})\in S_{n}^{k-1}$ sao cho $f(a,a_{2},\dots,a_{n})\neq f(b,a_{2},\dots,a_{n}),\ \forall a\neq b$. Khi đó, hiển nhiên tồn tại hàm song ánh $g$ trên $S_n$ sao cho: $$f(a,a_{2},\dots,a_{n})=g(a),\forall a\in S_{n}$$ Với bộ $(b_{2},b_{3},\dots,b_{n})\in S_{n}^{k-1}$ bất kì thỏa mãn $b_{i}\neq a_{i},\ \forall i=2,3,\dots,n$. Theo giả thiết: $$f(b_{1},b_{2},\dots,b_{n})\neq f(a,a_{2},\dots,a_{n})=g(a),\ \forall b_{1}\neq a$$ Từ đó, với chú ý $g$ là hàm song ánh, suy ra: $f(b_{1},b_{2},\dots,b_{n})=g(b_{1})$. Với bộ $(c_{2},c_{3},\dots,c_{n})\in S_{n}^{k-1}$ bất kì, vì $n>2$ nên tồn tại bộ $(b_{2},b_{3},\dots,b_{n})\in S_{n}^{k-1}$ thỏa mãn: $b_{i}\neq a_{i},c_{i}$ với mọi $i=2,3,\dots,n$. Áp dụng kết luận trên cho bộ $(a_{2},\dots,a_{n})$ và bộ $(b_{2},\dots,b_{n}$, ta có: $$f(b_{1},b_{2},\dots,b_{n})=g(b_{1})$$ Áp dụng kết luận đó lần nữa cho bộ $(b_{2},\dots,b_{n})$ và bộ $(c_{2},\dots,c_{n})$, ta có:

$$f(c_{1},c_{2},\dots,c_{n})=g(c_{1})$$ Bài toán được chứng minh trong trường hợp 1.

Trường hợp 2. Với mỗi bộ $(a_{2},a_{3},\dots,a_{k})\in S_{n}^{k-1}$, tồn tại hai số $a\neq {a}'$ sao cho: $$f(a,a_{2},\dots,a_{n})=f({a}',a_{2},\dots,a_{n})$$ Đặt hàm $h:S_{n}^{k-1} \mapsto S_{n}$ như sau: $h(a_{2},a_{3},\dots,a_{n})=f(a,a_{2},\dots,a_{n})=f(a',a_{2},\dots,a_{n})$, chú ý rằng nếu có nhiều cặp $(a,a')$ có tính chất như trên thì chọn một trong các cặp đó để định nghĩa cho hàm $h$. Dễ thấy, hàm $h$ thỏa mãn giả thiết quy nạp, nên tồn tại chỉ số $i$, không mất tổng quát ta giả sử $i=2$, và hàm song ánh $g$ trên $S_n$ thỏa mãn: $h(a_{2},a_{3},\dots,a_{n})=g(a_{2})$. Giả sử tồn tại bộ $(a_{1},a_{2},a_{3},\dots,a_{n})\in S_{n}^{k}$ sao cho: $f(a_{1},a_{2},\dots,a_n)\neq g(a_{2})$. Vì $g$ song ánh nên có $b_2\neq a_2$ sao cho: $$f(a_{1},a_{2},\dots,a_n) = g(b_{2})$$ Chú ý rằng $n>2$ nên chọn được bộ $(b_{3},\dots,b_{n})\in S_{n}^{k-2}$ sao cho $b_{i}\neq a_{i}$ với mọi $i=3,\dots,n$. Chọn $b_{1}\in \left \{ b,b' \right \}$ sao cho $b_{1}\neq a_{1}$, ở đây vai trò của $b,b'$ tương tự vai trò của của $a,a'$. Khi đó: $$f(b_{1},b_{2},\dots,b_{n})=h(b_{2},\dots,b_{n})=g(b_{2})=f(a_{1},a_{2},\dots,a_{n})$$ mâu thuẫn vì $a_{i}\neq b_{i},\ \forall i=1,2,\dots,n$. như vậy bài toán được chứng minh trong trường hợp 2.

Tóm lại bài toán được chứng minh hoàn toàn.




#427277 Phần trong $AD, BD, CD$ không chứa một điểm nào nguyên

Gửi bởi Trungpbc trong 14-06-2013 - 21:34

Gọi $L$ là tập các điểm nguyên trên mặt phẳng. Chứng minh rằng với mọi cặp $3$ điểm $A, B, C$ thuộc $L$ thì tồn tại điểm thứ tư $D$ sao cho phần trong của các đoạn thẳng (phần đoạn thẳng trừ đi $2$ đầu mút) $AD, BD, CD$ không chứa một điểm nào thuộc $L$.

Nhận xét:

Với hai điểm nguyên $X(a,b)$, $Y(c,d)$ thì đoạn thẳng $XY$ không chứa điểm nguyên nào ngoài hai đầu mút khi và chỉ khi $\gcd(a-c,b-d)=1$

Chứng minh nhận xét này đơn giản, xin không trình bày ở đây.

Trở lại giả bài toán ban đầu:

Giả sử $A(x_{1},y_{1}),B(x_{2},y_{2}),C(x_{3},y_{3})$, sử dụng nhận xét trên, ta thấy yêu cầu của bài toán tương đương với việc chứng minh tồn tại điểm nguyên $D(x,y)$ sao cho: $$\gcd(x-x_{i},y-y_{i})=1,\ \forall i=1,2,3$$ Dễ thấy, có tất cả $4$ điểm nguyên phân biệt theo modulo $2$ là $(0,0);(0,1);(1,0);(1,1)$. Do đó, có thể chọn $x,y$ theo modulo $2$ sao cho: $$(x,y)\neq (x_{i},y_{i})\pmod2,\ \forall i=1,2,3$$ Hoàn toàn tương tự, có thể chọn $x,y$ theo modulo $3$ sao cho: $(x,y)\neq (x_{i},y_{i})\pmod3,\ \forall i=1,2,3$. Theo định lí thặng dư trung hoa tồn tại số nguyên dương $x>\max\left \{ x_{1},x_{2},x_{3} \right \}$ thỏa mãn các điều kiện trên. Chú ý rằng  $T=(x-x_{1})(x-x_{2})(x-x_{3})>0$ nên $T$ có hữu hạn ước nguyên tố. Gọi $p>3$ là một ước nguyên tố bất kì của $T$.  Dễ thấy, có thể chọn $y$ modulo $p$ sao cho: $$y\neq y_{i}\pmod p,\ \forall i=1,2,3$$ Cho $p$ chạy trên tất cả các ước nguyên tố lớn hơn $3$ của $T$, sử dụng định lí thặng dư Trung Hoa, ta thấy tồn tại số nguyên dương $y$ thỏa mãn tất cả các điều kiện trên. Rõ ràng, điểm nguyên $D(x,y)$ thỏa mãn bài toán.




#415362 $\dfrac{1}{QM}+\dfrac{1}{QN...

Gửi bởi Trungpbc trong 29-04-2013 - 15:58

Lời giải của bạn robin997 không được ngắn gọn cho lắm, xin giới thiệu với các bạn một lời giải sáng sủa hơn.

BĐT đã cho tương đương với: $$\frac{QA}{QM.QA}+\frac{QB}{QN.QB}+\frac{QC}{QC.QP}\geqslant \frac{3}{R}$$ Chú ý, $QM.QA=QN.QB=QP.QC=R^{2}-QO^{2}$ nên ta có thể viết BĐT trên lại dưới dạng: $$QA.OA+QB.OB+QC.OC\geqslant 3(R^{2}-QO^{2})$$ Ta có: $$QA.OA\geqslant \overrightarrow{QA}.\overrightarrow{OA}=\left ( \overrightarrow{QO}+\overrightarrow{OA} \right )\overrightarrow{OA} \geqslant \overrightarrow{QO}.\overrightarrow{OA}+OA^{2}$$ Thiết lập các BĐT tương tự rồi cộng vế theo vế với chú ý: $$\overrightarrow{OA}+\overrightarrow{OB}+\overrightarrow{OC}=\overrightarrow{OH}=2\overrightarrow{OQ}$$ Suy ra: $$QA.OA+QB.OB+QC.OC\geqslant \overrightarrow{QO}.2\overrightarrow{OQ}+3R^{2}=3R^{2}-2OQ^{2}\geqslant 3(R^{2}-OQ^{2})$$ BĐT được chứng minh, đẳng thức xảy ra khi và chỉ khi tam giác $ABC$ là tam giác đều.




#409874 Cho một đa giác lồi, không có hai cạnh nào song song với nhau. Với mỗi cạnh c...

Gửi bởi Trungpbc trong 02-04-2013 - 10:16

Nhận xét của Heihitler hoàn toàn chính xác, có thể chỉ ra phản ví dụ trong lập luận của gogo123. (mong chú sớm post lời giải mới)

Để giải bài toán này ta cần sử dụng đặc trưng Minkowski của đa giác lồi.

Gọi $A_{1},A_{2},... ,A_{n}$ là các đỉnh của đa giác cần xét theo thứ tự ngược chiều kim đồng hồ và định hướng mặt phẳng (hướng dương) bởi hướng của đa giác $A_{1}A_{2},... A_{n}$. Chọn một điểm $O$ bất kì là gốc và vẽ $2n$ vector $\pm \overrightarrow{A_{i}A_{i+1}}$, $i=1,2,...,n$ với quy ước $A_{n+j}=A_{j}$. Gọi các vector gốc $O$ này theo thứ tự ngược chiều kim đồng hồ là $\overrightarrow{b_{1}},\overrightarrow{b_{2}},...,\overrightarrow{b_{2n}}$ (ở đây, vector $\overrightarrow{b_{1}}$ được chọn bất kì). Ta có một nhận xét rằng $\overrightarrow{b_{j}}=-\overrightarrow{b_{j+n}}$ với mọi $j=1,2,...,n$. Từ đó, ta xác định đặc trưng của $A_{1}A_{2},... A_{n}$ là hình $B_{1}B_{2}...B_{2n}$ bởi tính chất $\overrightarrow{B_{i}B_{i+1}}=\overrightarrow{b_{i}}$. Chú ý rằng $B_{1}B_{2}...B_{2n}$ là hình đóng do $\sum_{i=1}^{2n}\overrightarrow{b_{i}}=\overrightarrow{0}$ và là đa giác lồi do $\left (\overrightarrow{B_{i}B_{i+1}},\overrightarrow{B_{i+1}B_{i+2}}  \right )_{0} $ bé hơn $\pi $.

Rõ ràng, $\overrightarrow{B_{j}B_{j+1}}=-\overrightarrow{B_{j+n}B_{j+n+1}}$ với mọi $j$ nên tồn tại $O$ là tâm đối xứng của đa giác $B_{1}B_{2}...B_{2n}$

Xét $A_{i}A_{i+1}$ là một cạnh bất kì của đa giác ban đầu. Gọi $A_{k}$ là đỉnh có khoảng cách xa nhất đối với $A_{i}A_{i+1}$ và $j$ là chỉ số thỏa mãn $\overrightarrow{A_{i}A_{i+1}}=\overrightarrow{B_{j}B_{j+1}}$. Hiển nhiên: $$\overrightarrow{A_{i}A_{k}}=\overrightarrow{A_{i}A_{i+1}}+...+\overrightarrow{A_{k-1}A_{k}}=\overrightarrow{A_{i}A_{i-1}}+...+\overrightarrow{A_{k+1}A_{k}}$$ Và $$\overrightarrow{B_{j}B_{j+n}}=\overrightarrow{b_{j}}+...+\overrightarrow{b_{j+n-1}}$$ Chú ý $n$ vector $\overrightarrow{b_{j}},\overrightarrow{b_{j+1}},...,\overrightarrow{b_{j+n-1}}$ đôi một không cùng phương, góc định hướng $\left ( b_{j},b_{j+i} \right )_{0} <\pi $ với mọi $i=0,1,2,...,n-1$; đồng thời tất cả các vector đôi một không cùng phương và tạo với $\overrightarrow{A_{i}A_{i+1}}$ góc định hướng (với chu kì quay là $0$) nhỏ hơn $\pi$ là $\overrightarrow{A_{i}A_{i+1}},...,\overrightarrow{A_{k-1}A_{k}},\overrightarrow{A_{i}A_{i-1}},...,\overrightarrow{A_{k+1}A_{k}}$ suy ra $\overrightarrow{A_{i}A_{i+1}},...,\overrightarrow{A_{k-1}A_{k}},\overrightarrow{A_{i}A_{i-1}},...,\overrightarrow{A_{k+1}A_{k}}$ là một hoán vị của $\overrightarrow{b_{j}},\overrightarrow{b_{j+1}}...,\overrightarrow{b_{j+n-1}}$. Do đó: $$\overrightarrow{B_{j}B_{j+n}}=2\overrightarrow{A_{i}A_{k}}$$ Tương tự như vậy: $$\overrightarrow{B_{j+1}B_{j+n+1}}=2\overrightarrow{A_{i+1}A_{k}}$$ Suy ra $$\widehat{A_{i}A_{k}A_{i+1}}=\widehat{B_{j}OB_{j+1}}=\widehat{B_{j+n}OB_{j+n+1}}$$ Thành thử, tổng các góc cần tính chính bằng $$\frac{1}{2}\sum_{j=1}^{2n}\widehat{B_{j}OB_{j+1}}=\frac{360^{0}}{2}=180^{0}$$ ta có đpcm.




#407866 $ a_{mn} =a_m \cdot a_n;a_{m+n} \leq C...

Gửi bởi Trungpbc trong 25-03-2013 - 20:58

Đặt $f(n)=a_n$ để dễ trình bày, dễ thấy $f(0)=0$
Ta có: $f(n)$ là hàm nhân tính trên tập số nguyên dương; $f(1)=1;f(2)=2$
Suy ra: $f(2^{n})=2^{n}$ với mọi $n$.
Xét số nguyên dương $n$ bất kì, viết $n$ dưới dang $n= \sum_{i=0}^{k-1}a_i2^{i}$, với $a_i \in \left \{ 0;1 \right \} $.
Khi đó ta có: $3^{k-1}\leq n<3^{k}$.
Theo tính chất (2) của dãy ta có:
$$f(n)=f(\sum_{i=0}^{k-1}a_i2^{i})\leq C(\sum_{i=0}^{k-1}f(a_i)2^{i})\leq C(\sum_{i=0}^{k-1}2^{i})=C(2^{k}-1)<2Cn$$
Vậy $f(n)<2Cn$ với mọi n.
Do tính nhân tính của $f$ nên ta có
$(f(n))^{t}=f(n^{t})<2Cn^{t} \Rightarrow f(n)<\sqrt[t]{2C}n$
Cho $t \rightarrow + \infty $ thì $ \sqrt[t]{2C} \rightarrow 1$
Vậy với mọi $n$ ta có: $f(n) \leq n$
Bây giờ ta sẽ chứng minh $f(n)\geq n$ với mọi $n$.
Thật vậy: Vì $n<2^{k}$ nên tồn tại số nguyên dương $a$ sao cho $n+a=2^{k}$. Theo tính chất 2ta có:
$$f(n) \geq C(2^{k}-f(a))$$
Mặt khác theo tính chất đã chứng minh ở trên thì $f(a)\leq a \leq 2^{k}-2^{k-1}$
Suy ra: $f(n) \geq 2^{k}-(2^{k}-2^{k-1} \geq 2^{k-1} \geq \frac{nC}{2}$
Do đó : $(f(n))^{t}=f(n^{t}) \geq \frac{C}{2}.n^{t} \Rightarrow f(n) \geq \sqrt[t]{\frac{C}{2}}.n$
Tiếp tục cho $t \rightarrow + \infty $ ta có $f(n) \geq n$ với mọi $n$
Vậy ta có $f(n)=n$ với mọi $n$ hay $a_n=n$ (đpcm)

Ý tưởng trong lời giải của gogo123 khá độc đáo nhưng nếu để ý các bạn sẽ thấy nó đã phạm phải một sai lầm căn bản. Cụ thể ở chỗ mình bôi đỏ trên, đã thừa nhận tính chất: $$f\left ( n_{1}+n_{2}+\cdots +n_{k} \right )\leqslant C\left [ f(n_{1})+f(n_{2})+\cdots +f(n_{k}) \right ]$$ Chứng minh điều này không dễ và có lễ cũng là mấu chốt của bài toán. Dưới đây, ta sẽ chứng minh khẳng định mạnh hơn: $$f(x+y)\leqslant f(x)+f(y),\forall x,y\in \mathbb{N^{*}}$$ để rồi từ đó giải quyết bài toán rất nhẹ nhàng. Thật vậy, từ giả thiết, ta có: $$f(x+y+z)\leqslant C\left [ f(x)+f(y+z) \right ]\leqslant Cf(x)+C^{2}f(y)+C^{2}f(z)$$ Hoán đổi vai trò của $x,y,z$ rồi cộng theo vế các BĐT đó, ta được: $$f(x+y+z)\leqslant \frac{C+2C^{2}}{3}\left [ f(x)+f(y)+f(z) \right ]$$ Từ đó, với chú ý $f$ là hàm nhân tính và $f(2)=2$, suy ra:

$$ f(m+n)^{2}=f(m^{2}+2mn+n^{2})\leqslant \frac{C+2C^{2}}{3}\left [ f(m^{2})+f(2mn)+f(n^{2}) \right ]=\frac{C+2C^{2}}{3}\left [ f(m)+f(n) \right ]^{2} $$ Thành thử: $$ f(m+n)\leqslant \sqrt{\frac{C+2C^{2}}{3}}\left [ f(m)+f(n) \right ] $$ với mọi số nguyên dương $m,n$. Xét dãy $\left (c_{n}  \right ) $ được xác định như sau: $ c_{1}=C,c_{n+1}=\sqrt{\frac{c_{n}+2c_{n}^{2}}{3}} $. Dễ thấy, bằng quy nạp ta có được: $$ f(m+n)\leqslant c_{k}\left [ f(m)+f(n) \right ] $$ với mọi số nguyên dương $k$ và $c_{k}$ giảm dần về 1 khi $k$ dần ra vô cùng. Như vậy: $$ f(m+n)\leqslant f(m)+f(n) $$ với mọi số nguyên dương $m,n$. Một hệ quả dễ thấy là $f(n)\leqslant n$ với mọi số nguyên dương $n$.

Cố định một số nguyên dương $n$ bất kì và chọn số nguyên dương $k$ thỏa mãn $2^{k}>n$, khi đó: $$ 2^{k}=f(2^{k})\leqslant f(2^{k}-n)+f(n)\leqslant 2^{k}-n+n=2^{k} $$ Suy ra, dấu đẳng thức phải xảy ra, hay $ f(n)=n $. Phép chứng minh hoàn tất.




#384162 $|P(x)| \leq \frac{1}{2}, \forall x,...

Gửi bởi Trungpbc trong 06-01-2013 - 15:09

Cho $P(x) = x^{2} + ax+ b$. Biết rằng: $\forall x$ thỏa mãn $|x| \leq 1$, ta có $|P(x)| \leq \frac{1}{2}$
Tính giá trị của biểu thức $a^3 + b^3$

Lần lượt chọn $x=0,1,-1$ ta có:
$$\left\{\begin{matrix}
b\geqslant -1/2\\
a+b+1\leqslant 1/2\\
-a+b+1\leq 1/2
\end{matrix}\right.$$ Suy ra $\frac{-1}{2}\geqslant b\geqslant \frac{-1}{2}$ hay $b=\frac{-1}{2}$. Do đó, $a=0$.
Vậy $a^{3}+b^{3}=\frac{-1}{8}$


#381617 $f(p^m)$ chia hết cho $p$.

Gửi bởi Trungpbc trong 29-12-2012 - 17:36

Cho $a,b,c$ là các số nguyên, $b$ lẻ, xác định dãy $f(n), n=0,1,2,...$ như sau:
$$\left\{\begin{matrix} f(0)=4,f(1)=0,f(2)=2c,f(3)=3b\\ f(n+3)=af(n-1)+bf(n)+cf(n+1), \forall n \in \mathbb{N}^* \end{matrix}\right.$$
Chứng minh rằng với mọi số nguyên dương $m$, và mọi số nguyên tố $p$ ta có: $f(p^m)$ chia hết cho $p$.

DDTH

Phương trình đặc trưng của dãy $f(n)$ là $P(x)=x^{4}-cx^{2}-bx-a=0$, phương trình này có 4 nghiệm phức $\alpha _{i},i=1,2,3,4$ . Ta sẽ chỉ ra, phương trình này không có nghiệm bội.Giả sử phản chứng tồn tại số phức $\alpha $ sao cho $P(\alpha )=P'(\alpha )=0$, khi đó $\alpha $ là nghiệm của $$Q(x)=-4P(x)+xP'(x)=2cx^{2}+3bx+4a$$Là nghiệm của đa thức $$H(x)=2xQ(x)-cP'(x)=6bx^{2}+(4a+2c^{2})x+bc$$và là nghiệm của đa thức $$K(x)=cH(x)-3bQ(x)=(4ac+2c^{3}-9b^{2})x+(bc-12ab)$$ Như vậy, $\alpha $ là số hữu tỉ và là nghiệm của đa thức monic $P(x)$ nên là số nguyên. Khi đó, $P'(\alpha)$ là số lẻ, mâu thuẫn.
Từ đó, suy ra công thức tổng quát của $f(n)$ có dạng $$f(n)=a_{1}\alpha _{1}^{n}+a_{2}\alpha _{2}^{n}+a_{3}\alpha _{3}^{n}+a_{4}\alpha _{4}^{n}$$ Để ý $$f(0)=4=\alpha _{1}^{0}+\alpha _{2}^{0}+\alpha _{3}^{0}+\alpha _{4}^{0}$$ $$f(1)=0=\alpha _{1}+\alpha _{2}+\alpha _{3}+\alpha _{4}$$ $$f(2)=2c=-2\sum _{i<j}\alpha _{i}\alpha _{j}=\alpha _{1}^{2}+\alpha _{2}^{2}+\alpha _{3}^{2}+\alpha _{4}^{2}$$ $$f(3)=3b=3\sum _{i<j<k}\alpha _{i}\alpha _{j}\alpha _{k}=\alpha _{1}^{3}+\alpha _{2}^{3}+\alpha _{3}^{3}+\alpha _{4}^{3}$$ Suy ra, $a_{1}=a_{2}=a_{3}=a_{4}=1$ hay $$f(n)=\alpha _{1}^{n}+\alpha _{2}^{n}+\alpha _{3}^{n}+\alpha _{4}^{n}$$ Theo khai triển nhị thức Newton $$(x+y)^{p}=x^{p}+y^{p}+pQ(x,y)$$ với $Q(x,y)$ là một đa thức đối xứng hai biến $x,y$. Từ đó suy ra, $$(x+y+z+t)^{p}=x^{p}+y^{p}+z^{p}+t^{p}+pQ(x,y,z,t)$$ trong đó $Q(x,y,z,t)$ là đa thức đối xứng theo 4 biến $x,y,z,t$. Mặt khác, một đa thức đối xứng luôn có thể biểu diễn theo các đa thức đối xứng sơ cấp, cho nên $Q(\alpha_{1}^{p^{k}},\alpha_{2}^{p^{k}},\alpha_{3}^{p^{k}},\alpha_{4}^{p^{k}})$ là số nguyên với mọi số tự nhiên $k$. Suy ra
$$f(p^{m+1})=\left (\alpha _{1}^{p^{m}} \right )^{p}+\left (\alpha _{2}^{p^{m}} \right )^{p}+\left (\alpha _{3}^{p^{m}} \right )^{p}+\left (\alpha _{4}^{p^{m}} \right )^{p}=f(p^{m})^{p}-pQ(\alpha_{1}^{p^{m}},\alpha_{2}^{p^{m}},\alpha_{3}^{p^{m}},\alpha_{4}^{p^{m}})$$ chia hết cho $p$ khi và chỉ khi $p|f(p^{m})$. Từ đó với chú ý, $f(p^{0})=f(1)=0$ chia hết cho $p$, theo nguyên lí quy nạp, ta có đpcm.


#380668 Chứng minh tồn tại dãy số $(a_n)$, sao cho dãy $(b_n)$ vớ...

Gửi bởi Trungpbc trong 26-12-2012 - 20:25

Chọn dãy $a_n=(n+k+1)!+2,\forall n\in\mathbb{N}$, khi đó $b_n=(n+k+1)!+k+2$ chia hết cho $k+2$ với mọi $n$. Suy ra $b_n$ là hợp số với mọi $n$

Lời giải trên chưa đúng vì hiểu sai yêu cầu của bài toán. Bạn cần nhớ rằng dãy, $(a_{n})$ của ta cố định, trong khi $k$ là đại lượng thay đổi (chú ý trong đề bài có nhắc đến cụm từ "với mọi số nguyên dương $k$). Ta có thể giải đơn giản như sau bằng kiến thức về định lí thặng dư Trung Hoa.
Giả sử tập các số nguyên tố theo thứ tự tăng dần là $p_{0},p_{1},...$, khi đó với mọi số tự nhiên $i$, chọn $a_{i}$ thỏa mãn hệ phương trình đồng dư tuyến tính $$a_{i}\equiv -j \pmod{p_{j}},j=0,1,...,i$$ Rõ ràng mỗi hệ có vô số nghiệm nên có thể chọn sao cho $a_{i}>p_{i},\forall i\in \mathbb{N}$ và dãy của ta là dãy tăng thực sự các số nguyên dương. Để chứng minh dãy này thỏa mãn, gọi $k$ là một số tự nhiên bất kì, khi đó với tất cả các số tự nhiên $n\geqslant k$ thì $$a_{n}\equiv -k \pmod{p_{k}}$$ mà $a_{n}>p_{n}\geqslant p_{k}$ nên $a_{n}+k$ là hợp số với mọi $n\geqslant k$. Như vậy, dãy $(b_{n})$ chỉ chứa hữu hạn số nguyên tố. Từ đó, ta có đpcm.


#380207 Cho tứ giác $ABCD$ nội tiếp, $M$ là một điểm bất kì,...

Gửi bởi Trungpbc trong 25-12-2012 - 00:00

Cho tứ giác $ABCD$ nội tiếp, $M$ là một điểm bất kì, $X,Y,Z,T,U,V$ lần lượt là hình chiếu của $M$ lên các đường thẳng $AB,CD,AC,BD,AD,BC$. Gọi $E,F,G$ thứ tự là trung điểm của $XY,ZT,UV$ .Chứng minh rằng $E,F,G$ thẳng hàng.

Lời giải. Có ba khả năng xảy ra:
Khả năng 1. $M\equiv O$, khi đó $E,F,G$ trùng với trọng tâm $I$ của tứ giác $ABCD$, ta có đpcm.
Khả năng 2. $M\in \left ( O \right )$, khi đó, theo định lí về đường thẳng Simson ta có các bộ ba điểm$(X,Z,V); (V,Y,T); (Z,Y,U); (U,T,X)$ thẳng hàng nên $E,F,G$ là đường thẳng Gauss trong tứ giác toàn phần tạo bởi các điểm $X, Y, Z, T, U, V$. Và hiển nhiên ta có kết quả bài toán.
Khả năng 3. $M\notin \left ( O \right )$ và $M\not\equiv O$, gọi ${M}'\in OM\cap \left ( O \right )$ và ${X}',{Y}',{Z}',{T}',{U}',{V}'$ là hình chiếu của ${M}'$ trên các đường thẳng $AB,CD,AC,BD,AD,BC$. Vì $O,M,M'$ thẳng hàng nên các bộ ba điểm $(I,G,G'), (I,E,E'), (I,F,F')$ thẳng hàng và $\frac{\overline{IG}}{\overline{I{G}'}}=\frac{\overline{IE}}{\overline{I{E}'}}=\frac{\overline{IF}}{\overline{I{F}'}}$. Từ đó, với chú ý $E',F',G'$ thẳng hàng nên theo phép vị tự ta có $E,F,G$ thẳng hàng.


#379906 $\left| {{a_1} - 1} \right| = \left|...

Gửi bởi Trungpbc trong 23-12-2012 - 19:43

Cuối cùng cũng làm được bài tổng quát, chắc là sai sót vài chỗ không đáng kể. :icon6:

Không xét TH $a_i=i$, và chỉ xét TH số đang xét là số chẵn, tức là chỉ làm việc trên bộ $(1;2;...;2n)$
Đặt $d=\left | a_i-i \right | >0$.
Ta thấy $a_i=i+d$ hoặc $a_i=i-d$.
Suy ra 2 kết quả sau:
Nhận xét 1. $(a_i-i)(a_{i+2d}-(i+2d))>0$
GIả sử ngược lại thì $a_i=a_{i+2d}$, vô lí.
Nhận xét 2. $1\leq d\leq n$.
Thật vậy xét $a_n$ hoặc bằng $n+d$ hoặc bằng $n-d$ nên suy ra $n+d\leq 2n$ hoặc $n-d \geq 1$.Suy ra kết quả ở trên.

Theo nhận xét 1 ta có nếu $a_i=i+(-1)^{a}.d$ thì với mọi $j\equiv i$ $(mod$ $2d)$ thì $a_j=j+(-1)^{a}.d$.
Như vậy với mỗi $d$ ta chia tập ${1;2;...;2n}$ thành các tập rời rạc đồng dư nhau modun $2d$ , khi đó các phần tử cùng một tập thì có chung dấu trước $d$ (là $-$ hay $+$)
Với mỗi $d$ thì $a_i=i+d$ với mọi $1\leq i\leq d$ vì nếu ngược lại thì có $a_i<0$ là điều vô lí,và $a_i=i-d$ với mọi $d+1\leq i\leq 2d$ vì nếu ngược lại thì tập $a_i$ không chứa các phần tử ${1;2;...;d}$,vô lí.
Vậy với mỗi $d$ chỉ có đúng một hoán vị thõa mãn.
Mặt khác theo nhận xét 2 thì $d \in {1;2;...;n}$ thõa mãn.Cộng thêm TH hoán vị đồng nhất ta đã không xét.
Vậy có tất cả $n+1$ hoán vị thõa mãn. (Chú ý ta đang xét TH tập ${1;2;...;2n}$ )


:icon10: Đáp án tổng quát: $\left [\frac{n}{2} \right ]+1$ :icon10:

Mình thấy đấp số này chưa đúng, chẳng hạn với kết quả tổng quát, cho $n=3$ thì ta chỉ có hoán vị tầm thường $a_{i}=i$ thỏa mãn. Trường hợp tổng quát, mình làm được số các hoán vị thỏa mãn tính chất là $1+d(\frac{n}{2})$ nếu $n$ chẵn và $1$ nếu $n$ lẻ, ở đây $d(m)$ là hàm tính số các ước nguyên dương của $m$


#369435 $f: \mathbb{R}^+ \to \mathbb{R}^+$

Gửi bởi Trungpbc trong 14-11-2012 - 18:00

Xác định hàm số $f: \mathbb{R}^+ \to \mathbb{R}^+$ thoả mãn các điều kiện
a- $f(2)=0$
b- $f(x) \ne 0$
c- $f[x.f(y)].f(y)=f(x+y) \,\, \forall x,y \in \mathbb{R}^+$
M.n giúp giùm pài này nhanh xíu mình đang gấp nha. :D

$\mathbb{R}^+$ là tập các số thực dương (không chứa $0$) nhưng trong bài toán này, ta phải hiểu nó có chứa phần tử $0$ (đáng lẽ trong đề bài cần có chú thích rõ đoạn này) vì đây chính là problem 5 IMO 1986. Sau đây mình xin trình bày lời giải một mở rộng đẹp của bài toán IMO 1986 đó.
Bài toán. Kí hiệu $\mathbb{R}_{0}^{+}$ để chỉ tập các số thực không âm. Xác định tất cả các hàm $f:\mathbb{R}_{0}^{+}\rightarrow \mathbb{R}_{0}^{+}$ thỏa mãn $$f(x)f(yf(x))=f(x+y)$$ với mọi $x,y\in \mathbb{R}_{0}^{+}$.
Lời giải. Chọn $y=0$ ta có $f(x)f(0)=f(x)$. Bỏ qua trường hợp tầm thường $f(x)=0,\forall x\in \mathbb{R}_{0}^{+}$, suy ra $f(0)=1$. Bây giờ, xét hai khả năng có thể xảy ra:
Khả năng 1. Tồn tại $x>0$ sao cho $f(x)=0$, đặt $a=\inf\left \{ x|f(x)=0,x\in \mathbb{R}_{0}^{+} \right \}$ (chú ý, có được điều này là bởi vì, mọi $x$ thỏa mãn như vậy đều bị chặn dưới bởi $0$). Theo định nghĩa cận dưới đúng, với mọi $y>a$, tồn tại $x$ thỏa mãn $y>x\geqslant a$ và $f(x)=0$. Khi đó, ta có: $$f(y)=f(y-x+x)=f(x)f((y-x)f(x))=0$$ Do đó, với mọi $x\in (a,+\infty)$ thì $f(x)=0$. Nếu $a=0$ thì ta thu được hàm số $$f(x)=\left\{\begin{matrix}
1 &,x=0 \\
0&,x>0
\end{matrix}\right.$$ Việc kiểm tra hàm số này thỏa mãn khá đơn giản, không trình bày ở đây.
Xét trường hợp còn lại: $a>0$, theo định nghĩa của $a$ thì $f(x)>0,\forall x<a$. Mặt khác, với mọi $\varepsilon >0$ và $x<a$, ta có: $$0=f(a+\varepsilon )=f(x)f((a+\varepsilon -x)f(x))$$ Suy ra $$f((a+\varepsilon -x)f(x))=0$$ Cho nên, từ định nghĩa của $a$, ta phải có $$(a+\varepsilon -x)f(x)\geqslant a$$ Hệ quả là $f(x)>0,\forall x<a$ và $f(x)\geqslant \frac{a}{a+\varepsilon -x},\forall \varepsilon >0$. Cố định $x$, cố định $x$ cho $\varepsilon \rightarrow 0^{+}$, thu được kết quả $$f(x)\geqslant \frac{a}{a-x},\forall x<a$$ Lại có $$f\left ( x+\frac{a+\varepsilon }{f(x)} \right )=f(x)f\left ( \frac{a+\varepsilon }{f(x)}.f(x) \right )=0$$ nên $ x+\frac{a+\varepsilon }{f(x)} \geqslant a,\forall \varepsilon >0$. Có định $x$, cho $\varepsilon \rightarrow 0^{+}$, ta có $$f(x)\leqslant \frac{a}{a-x},\forall x<a$$ Do đó, $$f(x)=\frac{a}{a-x},\forall x<a$$ Đặc biệt, chọn $x=\frac{a}{2}$ thì $f\left ( \frac{a}{2} \right )=2$. Cho $x=y=\frac{a}{2}$ vào phương trình ban đầu, ta có $f(a)=0$. Tóm lại, hàm $f$ trong TH này được xác định như sau $$f(x)=\left\{\begin{matrix}
\frac{a}{a-x}&,x< a \\
0 & ,x\geqslant a
\end{matrix}\right.$$ Kiểm tra hàm này thỏa mãn bài toán đơn giản không trình bày ở đây.
Khả năng 2. Không tồn tại $x>0$ sao cho $f(x)=0$, nói cách khác $f(x)>0$ với mọi $x>0$. Các bạn có thể tham khảo lời giải của perfecstrong. Ngoài ra còn một lời giải bằng đạo hàm (khá ngắn gọn) trong TH này, bạn nào quan tâm thì suy nghĩ tiếp xem nhé!


#368341 $\sum\limits_{k=0}^{n}(-1)^kC_n^k(x-k)^n=n...

Gửi bởi Trungpbc trong 10-11-2012 - 08:23

Ta sẽ chứng minh khẳng định tổng quát hơn.
Cho đa thức: $$f(X)=a_{n}X^{n}+a_{n-1}X^{n-1}+...+a_{1}X+a_{0}$$ Khi đó với mọi số thực $h$ khác $0$ và mọi số thực $x$, ta có đẳng thức $$\sum_{k=0}^{n}(-1)^{n-k}\binom{n}{k}f(x+kh)=a_{n}.n!.h^{n}$$

Đẳng thức trên không hề tầm thường, nó là một trong những bổ đề quan trọng giải quyết những bài đa thức khó. Tiêu biểu trong số đó là bài toán sau:
Bài toán. Cho hai đa thức $P,Q$ cới hệ số nguyên sao cho với mọi số nguyên dương $n$, $P(n), Q(n)$ là những số nguyên dương thỏa mãn $$2^{Q(n)}-1|3^{P(n)}-1$$Chứng minh rằng $Q$ là đa thức hằng.
Lời giải.
Cố định một số nguyên dương $n$, đặt $a=o_{2^{Q(n)-1}}(3)$ (kí hiệu $o_{p}(x)$ để chỉ bậc của $x$ modulo $p$). Từ giả thiết suy ra $$3^{P(n)}-1\equiv 0\pmod{2^{Q(n)}-1}$$ Suy ra $a|P(n)$. Mặt khác với mọi số nguyên dương $k$ thì ta có $Q(n+kQ(n))\equiv 0\pmod{Q(n)}$. Do đó, kết hợp với giả thiết cho ta $$3^{P(n+kQ(n))}\equiv 1\pmod{2^{Q(n)}-1}$$ Suy ra $$P(n+kQ(n))\equiv 0\pmod a,\forall k\in \mathbb{N}$$ Từ đó, chọn $m=\deg P$, ta có $$a|\sum_{k=0}^{m}(-1)^{m-k}\binom{m}{k}P(n+kQ(n))=p.Q(n)^{m}m!$$ trong đó, $p$ là hệ số bậc cao nhất của $P$. Suy ra, $a|\gcd\left (P(n);p.m!Q(n)^{k} \right )$. Mặt khác, do hai đa thức $P,Q$ nguyên tố cùng nhau nên theo định lí Bezout, tồn tại số nguyên dương $A$ và hai đa thức $S,R$ với hệ số nguyên sao cho $$P(x).S(x)+Q(x).R(x)=A$$ Hệ quả là, $\gcd(P(n),Q(n))|A $ hay $\gcd(P(n),Q(n))$ bị chặn. Và như thế, $\gcd(P(n),p.m!Q(n)^{k})$ bị chặn hay $a=o_{2^{Q(n)}-1}(3)$ bị chặn. Suy ra $2^{Q(n)}-1|3^{a}-1$ bị chặn. Điều này chứng tỏ, $Q$ phải là hằng số.